Finding a Basis for the Nullspace of a 2x2 Matrix Transformation

  • Thread starter Thread starter charlies1902
  • Start date Start date
  • Tags Tags
    Basis Nullspace
Click For Summary
To find a basis for the nullspace of a 2x2 matrix transformation, it is essential to identify which matrices are transformed to the zero matrix. The discussion reveals confusion between the nullspace and the range of the transformation. The user initially considers matrices that appear to form a linear combination but struggles to reconcile this with the book's answer. Ultimately, the correct basis for the nullspace is identified as the matrices that yield the zero matrix when the transformation is applied. Understanding that the nullspace is determined by setting the transformation equal to zero clarifies the distinction between nullspace and range.
charlies1902
Messages
162
Reaction score
0
The problem is attached.

I am instructed to find a basis for the nullspace of T.A basis for a 2x2 matrix is
1 0
0 0

0 1
0 0

0 0
1 0

0 0
0 1Applying the transformation to each of these gives
0 0
0 0

0 2
0 0

0 0
-2 0

0 0
0 0
respectively.

Now this is where I get stuck. How do I find a basis after knowing these 4 matrices?

It look like it's a linear combo of 2 matrices:
0 1
0 0
and
0 0
1 0

but the answer in the book is
1 0
0 0

0 0
0 1

I don't undertand.
 

Attachments

  • Untitled.png
    Untitled.png
    2.4 KB · Views: 461
Last edited:
Physics news on Phys.org
charlies1902 said:
The problem is attached.

I am instructed to find a basis for the nullspace of T.A basis for a 2x2 matrix is
1 0
0 0

0 1
0 0

0 0
1 0

0 0
0 1Applying the transformation to each of these gives
0 0
0 0

0 2
0 0

0 0
-2 0

0 0
0 0
respectively.

Now this is where I get stuck. How do I find a basis after knowing these 4 matrices?

It look like it's a linear combo of 2 matrices:
0 1
0 0
and
0 0
1 0

but the answer in the book is
1 0
0 0

0 0
0 1

I don't undertand.

Which matrices get sent to (transformed to) the zero matrix? That's the basis for Null(T).
 
Gotcha, I keep it getting it cnofused with range (T), nullspace(T) is solved by setting the right hand side of the transformation equal to 0 right?So for this case,
0 1
0 0
and
0 0
1 0

would be a basis for the range of T?
 
I think so, but I didn't check your work.
 
Question: A clock's minute hand has length 4 and its hour hand has length 3. What is the distance between the tips at the moment when it is increasing most rapidly?(Putnam Exam Question) Answer: Making assumption that both the hands moves at constant angular velocities, the answer is ## \sqrt{7} .## But don't you think this assumption is somewhat doubtful and wrong?

Similar threads

  • · Replies 2 ·
Replies
2
Views
1K
  • · Replies 14 ·
Replies
14
Views
3K
Replies
8
Views
2K
Replies
9
Views
2K
  • · Replies 2 ·
Replies
2
Views
2K
  • · Replies 8 ·
Replies
8
Views
2K
  • · Replies 24 ·
Replies
24
Views
4K
  • · Replies 1 ·
Replies
1
Views
2K
  • · Replies 2 ·
Replies
2
Views
2K
  • · Replies 3 ·
Replies
3
Views
2K